Sunteți pe pagina 1din 17

CICLO PRLUNIVLRSI1ARIO ADMISIN 2007-I SLMINARIO N 02

CLPRL-UNI ISICA 1
$

01. Seale la veracidad (V) o falsedad (F)
de las siguientes proposiciones:
. Un objeto necesariamente se
encontrar en reposo si sobre el no
acta fuerza alguna.
. En ausencia de fuerza externa un
objeto en movimiento continuar en
movimiento con velocidad
constante.
. Aquello que puede alterar el estado
de movimiento de un objeto se
denomina fuerza.
A) VVV B) FVV C) FFV
D) FFF E) VFF

02. Seale la veracidad (V) o falsedad (F)
de las siguientes proposiciones
respecto a la primera ley de Newton:
. Si un cuerpo en movimiento, en
ciertos instantes est en reposo, en
dichos instantes el cuerpo se
encuentra en equilibrio.
. Todo cambio en la velocidad de un
cuerpo indica el efecto de una
fuerza.
. Si el velocmetro de un cuerpo
indica rapidez constante entonces
la fuerza neta sobre dicho cuerpo
es cero.
A) VVF B) FVF C) VVV
D) FVV E) VFF

03. Respecto al concepto de fuerza,
indique la veracidad (V) o falsedad (F)
de las siguientes proposiciones:
. Surge cuando dos o ms cuerpos
se encuentran adyacentes.
. Es una cantidad escalar y la unidad
en el S.. es el newton (N).
. Los cuerpos que interactan,
necesariamente tienen que estar
adyacentes.
A) VVV B) VVF C) VFV
D) VFF E) FFF

04. Un objeto es soltado por un riso
circular liso, desde la posicin "A, el
D.C.L. ms adecuado del objeto en la
posicin "B es:













A) B) C)





D) E)

05. Jaimito, quien se encuentra parado en
el centro de la plataforma, est en
reposo. Si Jaimito tiene masa m y la
plataforma tiene masa M, cules de
los siguientes DCL son correctos?.









. .







R
B
A
N
F

F

N
F

C F
F

F

N
F

c F
F

0 0
T
1

T
1

1
mg
2

Mg
1
T 3
mg

F

c F
F

N
F

CICLO PRLUNIVLRSI1ARIO ADMISIN 2007-I SLMINARIO N 02

CLPRL-UNI ISICA 2
.





A) Solo B) Solo C) Solo
D) Todos E) Ninguno

0. Para que el cuerpo mostrado en la
figura, sea considerado como una
partcula en equilibrio cul(es) de las
siguientes condiciones es (son)
necesaria(s)?
. 1 2 3 F F F 0


. 1 2 3 F , F y F

son colineales
. 1 2 3 F , F y F

son concurrentes.








A) Solo B) Solo C) Solo
D) Solo y E) Solo y

07. El sistema mostrado se mueve con
velocidad constante v 4i

m/s; las
cuerdas no tienen una masa
significativa y la del bloque es .
Halle lo que marca el dinammetro
ideal "D (en N) g 10 m/s
2
.









A) 30 B) 0 C) 30 3
D) 0 3 E) 120
08. Halle la tensin en el cable (1) del
sistema en equilibrio mostrado en la
figura.

A)
1
2
mg
B) mg
C)
3
2
mg
D) 2 mg
E)
5
2
mg

09. Si se sabe que el peso de A es de
200 N, calcule la suma de los pesos
(en N) de B; C y D.










A) 250 B) 280,5 C) 300,2
D) 33, E) 358,1

10. En la figura se muestran dos esferas
iguales de radio R 0,2 m y masa
. Calcule la reaccin de B sobre
A (en N) asumiendo que todas las
superficies son lisas.







A)
40
3
3
B) 20 3 C)
20
3
3

D) 30 2 E)
20
2
3

T
1
T
1

Mg
mg
2 m
m
(1)
g


0
A
B
0
D
0
v


2 F


1 F

3
F


0
A
B
0
30
C
D
CICLO PRLUNIVLRSI1ARIO ADMISIN 2007-I SLMINARIO N 02

CLPRL-UNI ISICA 3
11. Dos cilindros de masas 10 kg se
encuentran en reposo como se
muestra en la figura. Suponiendo que
todos los contactos son lisos, halle la
fuerza F (en N) con que el resorte
sostiene al cilindro.

A) 24
B) 3
C) 48
D) 0
E) 72





12. La piedra triangular issceles
homognea de encaja en la
pared vertical mantenindose en
equilibrio con la fuerza
F 300i 500j N


. Si los contactos
son lisos, halle la fuerza de reaccin
(N) sobre la piedra en la superficie
horizontal.









A) 700 j

B) 300 j

C) 500 j


D) 800 j

E) 800 j



13. Respecto al torque, indique la
veracidad (V) o falsedad (F) de las
siguientes proposiciones:
. Es una cantidad escalar y su
unidad en el S.. es mN.
. El torque sobre un cuerpo solo
depende de la fuerza que acta
sobre el.
. El torque es perpendicular al plano
formado por el brazo de palanca y
la fuerza.
A) VVV B) VVF C) VFV
D) FFV E) FVF

14. La placa es un hexgono regular de
de lado. De las 3 fuerzas
mostradas, calcule la magnitud del
torque resultante (en N.m) respecto del
punto A.











A) 5,45 B) 4,20 C) 3,
D) 2,80 E) 1,50

15. La figura muestra la varilla AOB al que
se le aplican fuerzas, si la varilla tiene
masa insignificante, determine el
torque resultante (en N.m) debido a las
dos fuerzas respecto del punto O.










A)

25k B)

25k C)

30k
D)

30k E)

50k




T
127 k
y
x
z
A
10 N
5 N
y

x

m
z

A

15 N

O

3 m
B

20 N

g


pared

vertical

x
y -
-
CICLO PRLUNIVLRSI1ARIO ADMISIN 2007-I SLMINARIO N 02

CLPRL-UNI ISICA 4
1. La barra homognea mostrada pesa
140 N y el torque respecto de O es

24k Nm cuando se aplica sobre B


una fuerza F

cuya direccin est a lo


largo del eje +Y. Evalu el torque de F


respecto de A. Calcule








A)

18k B)

18k C)

70k
D)

70k E) 100 j



17. La varilla doblada de la figura puede
girar en torno a O y es de masa
insignificante. Calcule en N.m el
mdulo del torque de F

respecto a O
si F

.









A) 0 B)
F
3

C)
F
2


D) F E)
F
3



18. La barra homognea AB de masa 9 kg
est sostenida por cuerdas (1) y (2).
Halle el torque de la tensin en (1)
respecto de A. Considere
AB
L
y
2
g 10 m/s .











A)

50k B)

30k C)

20k
D)

40k E)

0k

19. ndique la veracidad (V) o falsedad (F)
de las proposiciones siguientes:
. Si un cuerpo rgido se encuentra en
equilibrio, la suma de los torques de
todas las fuerzas que sobre l
actan respecto a cualquier punto,
es cero.
. Si la resultante de todas las fuerzas
que actan sobre un cuerpo rgido
es cero, entonces est en equilibrio.
. Si un cuerpo rgido no se encuentra
en equilibrio, necesariamente la
suma de los momentos de todas las
fuerzas que actan sobre l es
diferente de cero.
A) VVV B) FFF C) VFF
D) VVF E) FFV

20. La figura muestra una esfera
homognea de masa M y radio R. Si a
sta se aplica una fuerza F

horizontal,
como indica la figura y las superficies
son lisas, seale la veracidad (V) o
falsedad (F) de las siguientes
proposiciones:
. Para cualquier valor de F

(siempre
que no se rompa la esfera o la
pared, la normal ejercida por el piso
tiene igual mdulo que el peso).
. Para un F

relativamente grande la
normal del piso vale cero.
. El torque resultante respecto al
centro de la esfera es igual al
y

x

4 m
B
A
143
O
g


A
(1)
1 m
(2)
B
y

x

z

g


3

O
CICLO PRLUNIVLRSI1ARIO ADMISIN 2007-I SLMINARIO N 02

CLPRL-UNI ISICA 5
torque resultante respecto al punto
de apoyo en el piso.









A) VFV B) VVV C) FVF
D) FFV E) FFF

21. La esfera de radio R y la barra
homognea de longitud 4R de la figura
tienen pesos iguales estn en
equilibrio. Halle el ngulo 7 si la cuerda
que sostiene a la esfera mide R.








A)

1
tg 2 3

B)

1
tg 2 3


C)

1
sen 3 2

D)
1
tg 3


E)
1
5
sen
2



22. Una barra de peso y longitud " se
encuentra en equilibrio apoyndose
sobre una pared vertical lisa y un piso
horizontal rugoso. Determine la
reaccin del piso sobre la barra.









A)
1/ 2
2

1 cot g
2
7
B)

1/ 2
2

1 sec
2
7
C)
1/ 2
2

4 t g
2
7
D)

1/ 2
2

4 sen
2
7
E)
1/ 2
2

4 cot g
2
7

23. El peso de la barra homognea en la
figura es de y su longitud es
2 m. Cul es la tensin (en N) de la
cuerda?


A) 10
B) 20
C) 50
D) 80
E) 100



24. La figura muestra una varilla
homognea que se encuentra apoyada
en una bscula permaneciendo en
reposo bajo la accin de la fuerza F


(perpendicular a la varilla). Si la lectura
de la balanza es de , determine
la masa (en kg) de la varilla.

2
g 9,8 m/s .


A) 1,02
B) 2,01
C) 3,05
D) 5,10
E) ,54



7
30
F


53
7
CICLO PRLUNIVLRSI1ARIO ADMISIN 2007-I SLMINARIO N 02

CLPRL-UNI ISICA 6
25. Hallar el mdulo de la fuerza "F para
que la barra de 1 kg, permanezca
horizontal ( 10 N ).










A) 5 B) 10 C) 15
D) 20 E) 25

2. La barra uniforme AB de de
longitud mostrada en la figura, tiene
una masa de 180 kg. Si todas las
superficies son lisas, determine la
mayor fuerza de reaccin (en N) sobre
la barra. Considere a la cuerda CD de
masa insignificante.










A) 200 B) 300 C) 400
D) 500 E) 00

27. En el sistema mostrado no existe
friccin, el bloque A recibe una
reaccin de de la superficie
inclinada si la polea P tiene peso
insignificante y la polea Q pesa 10 N y
considerando que el sistema se
encuentra en equilibrio, determine los
pesos de los bloques A y B (en N).












A) 100 y 25 B) 120 y 30
C) 120 y 50 D) 150 y 50
E) 180 y 0

28. El bloque de masa m mostrado en la
figura se mantiene fijo al aplicarse una
fuerza F mediante el sistema de
poleas y cuerdas ideales. Determine la
tensin T, en la cuerda mostrada.


A) mg/2
B) mg
C) 3mg/2
D) 2mg
E) 5mg/2



29. Con relacin a las siguientes
proposiciones sobre las fuerzas de
friccin indique verdadero (V) o falso
(F).
. Siempre actan tangentes a la
superficie en contacto.
. No dependen del rea de las
superficies en contacto.
. Para dos superficies determinadas
la fuerza de rozamiento esttico es
mayor que la fuerza de rozamiento
cintico.
A) VVV B) VVF C) VFV
D) FVV E) FFV




C
D
A
B
12 m
P
Q
B
m

T


F


g

F

20 cm 80 cm

g


CICLO PRLUNIVLRSI1ARIO ADMISIN 2007-I SLMINARIO N 02

CLPRL-UNI ISICA 7
30. Respecto de las fuerzas de
rozamiento, indique la veracidad (V) o
falsedad (F) de las proposiciones
siguientes:
. Siempre es contraria al movimiento
del cuerpo sobre el que acta.
. Todas las fuerzas de rozamiento
desgarran o desgastan las
superficies sobre las que actan.
. La magnitud del rozamiento o
friccin esttico, es siempre
proporcional a la fuerza normal de
contacto a las superficies sobre los
que actan.
A) VVV B) FFF C) VFV
D) FFV E) FVF

31. En la figura todos los contactos tiene
friccin esttica 3 . Si el
sistema se encuentra en reposo,
calcule el mximo valor del peso (en
N) del bloque de la masa M si
m 2 kg .










A) 20 B) 40 C) 0
D) 80 E) 100

32. Se muestra los instantes en que un
bloque lanzado, pasa por dos
superficies (1), (2) con las velocidades
indicadas. Si los coeficientes de
rozamiento del bloque con cada
superficie valen
1
3 y
2
3 ;
respectivamente, para qu ngulo 7
las fuerzas de friccin respectivas
tendrn el mismo mdulo?








A)
1
2
arc sen
3

3

B)
1
2
arc cos
3

3


C)
1
2
arc tag
3

3

D)
2
1
arc sen
3

3


E)
2
1
arc cos
3

3



33. En el aeropuerto Jorge Chvez se
desea instalar una faja para el
transporte de las maletas desde las
bodegas del avin hasta el carrito que
las lleva al almacn. Si la faja tiene
una inclinacin de 30 respecto de la
horizontal, y transporta las maletas con
velocidad constante, el valor del
coeficiente de rozamiento esttico que
debe haber entre las maletas y la faja,
para que stas no deslicen debe ser
mayor que:
A) 0,57 B) 0,0 C) 0,3
D) 0,70 E) 0,78

34. Sobre la segunda ley de Newton,
indique las proposiciones verdaderas
(V) o falsas (F):
. Es una trayectoria curva la
direccin de la aceleracin es
diferente al de la fuerza.
. En la segunda ley de Newton no se
cumple el principio de
superposicin de fuerzas.
. Las fuerzas a que se refiere la
segunda ley de Newton son fuerzas
internas del sistema analizado.
A) VVV B) VVF C) VFF
D) FVV E) FFF

2m

M

m

7
(1)

(2)

1 m/s

5 m/s

CICLO PRLUNIVLRSI1ARIO ADMISIN 2007-I SLMINARIO N 02

CLPRL-UNI ISICA 8
35. Sobre un cuerpo de 2 kg actan, una
fuerza de y otra fuerza
perpendicular a la anterior. Si el
mdulo de la aceleracin del cuerpo
debido a estas dos fuerzas es de
2
2,5 m/s . Encontrar la fuerza
perpendicular expresada en newtons.
A) 2 B) 3 C) 4
D) 5 E)

3. Determine la mxima aceleracin que
un atleta de masa "m puede alcanzar,
sobre una superficie horizontal
sabiendo que el coeficiente de
rozamiento esttico entre las
superficies en contacto es 3.





A)
g
3
B)
g
3
C)
g
1
3

D) g 3 E)
2
g 3

37. Un bloque sube por un plano inclinado,
que hace un ngulo de 37 con la
horizontal, luego de ser lanzado con
velocidad inicial de . Cunto
tiempo en segundos tarda en alcanzar
su mxima altura, si el coeficiente de
rozamiento entre el bloque y el plano
es 0,3?
2
g 9,8 m/s .
A) 1,51 B) 1,82 C) 2,53
D) 4,25 E) ,25

38. En el sistema mostrado el ascensor
desciende con una aceleracin
2
a 2j m/s

. Si la masa m 10 kg y el
hombre de masa M se
encuentran en reposo respecto del
ascensor, determine la reaccin (en N)
del piso sobre el hombre.




A) 320
B) 490
C) 50
D) 00
E) 50


39. Si la partcula (2) de la figura se mueve
con
2
m
a 2,5
s
, halle -.









A) 15 B) 30 C) 45
D) 0 E) 90

40. En la figura el coeficiente de
rozamiento cintico entre los bloques
de 2 kg y 3 kg es . No hay
rozamiento en la superficie horizontal y
en las poleas.
a) Dibuje el D.C.L. de cada bloque.
b) Halle la magnitud de la aceleracin
(en m/s
2
) con que se desplaza el
bloque de 2 kg.










A) 2,3 B) 3,2 C) 4,8
D) 5,9 E) 7,0
a


-
m
m
(1)
(2)
2
g 10 m/s


liso
liso
3 kg



10 kg
2 kg
CICLO PRLUNIVLRSI1ARIO ADMISIN 2007-I SLMINARIO N 02

CLPRL-UNI ISICA 9
41. Un bloque de descansa sobre
otro de 0,8 kg. El conjunto es
arrastrado a velocidad constante sobre
una superficie horizontal por un
bloque de masa 0,2 kg, como se
muestra en la figura (1).
Posteriormente el primer bloque de
0,2 kg se separa del de 0,8 kg y se
une al bloque suspendido como en la
figura (2). cul es la aceleracin (en
m/s
2
) del bloque de 0,8 kg en estas
condiciones?. (g = 10 m/s
2
).








A) 1 B) 2 C) 3
D) 4 E) 5

42. ndique las proposiciones verdaderas
respecto de una partcula que se
mueve en trayectoria circular.
. Si su velocidad angular es
constante entonces su aceleracin
centrpeta es constante.
. Si su aceleracin angular es
constante y mayor que cero
entonces su aceleracin tangencial
es de mdulo constante.
. La velocidad angular y la
aceleracin angular tienen siempre
la misma direccin y sentido.
A) Solo B) Solo C) Solo
D) Solo y E) Solo y

43. Un cuerpo de masa m 2kg se mueve
con MCUV de radio R 2m . Si la
magnitud de la fuerza centrpeta en el
punto A es
cA
F y la rapidez en
el punto B es
B
v 10 m/s, determine
la rapidez (en m/s) cuando el cuerpo
pase nuevamente por A.









A) 37, B) 19,7 C) 18,4
D) 1,4 E) 3,1

44. Un carro que se mueve en una
montaa rusa tiene una masa de
00 kg cuando est totalmente lleno
de pasajeros. Si el vehculo tiene una
rapidez de 20 m/s en el punto A. Cul
es la fuerza ejercida (en N) por la pista
sobre el vehculo en ese punto? Cul
es la rapidez mxima (en m/s) que el
carro puede tener en B y continuar
sobre la pista?







A) 25000 , 14 B) 20000 , 14
C) 30000 , 12,2 D) 25000 , 12,2
E) 24000 , 12,2

45. Un disco, de radio R 50cm , gira con
una velocidad angular igual a
10 rad/s (vase la figura). A qu
distancia mxima (en cm) del centro
del disco debera estar un objeto para
que gire con el sin deslizar, considere
s
0, 2 3 ?

A) 1
B) 17
C) 18
D) 19
E) 20

B
A
0,8 kg 0,8 kg
0,2 kg
Figura (1) Figura (2)
0,2 kg
0,2 kg
0,2 kg
g



A
B
R


r
CICLO PRLUNIVLRSI1ARIO ADMISIN 2007-I SLMINARIO N 02

CLPRL-UNI ISICA 10
4. La rampa en forma de cono gira con
velocidad angular 0,5 rad/s . Si el
bloque gira con el cono sin resbalar
pero en movimiento relativo inminente,
halle el coeficiente de friccin bloque-
cono.






A)
30
37
B)
31
37
C)
32
37

D)
33
37
E)
34
37


47. ndique cules de los siguientes
sistemas no son inerciales?
. Una nave espacial que despega
con aceleracin constante respecto
de tierra.
. Una persona que se mueve con
aceleracin nula sobre una pista.
. Un automvil que se desplaza con
rapidez constante en una pista
circular.
A) Solo B) Solo C) Solo
D) Solo y E) Solo y

48. ndique cul de las siguientes
proposiciones son correctas respecto
de sistemas de referencias inerciales
(S.R)?
. La velocidad de un auto respecto
de un S.R(1) fijo a tierra es v

. Para
otro S.R(2) el auto tiene velocidad
cero, entonces este S.R se mueve
con velocidad v

respecto del
S.R.(1).
. Dos S.R., (1) fijo a tierra y (2) con
velocidad constante horizontal
respecto de (1), observan el
movimiento de un proyectil. Para
ambos sistemas el peso del
proyectil es la nica fuerza que
acta sobre l.
. En el caso () se puede afirmar que
para el S.R.() la trayectoria del
proyectil es una parbola y para el
S.R(2) la trayectoria es una lnea
recta (proyectil en cada libre).
A) Solo B) Solo C) Solo
D) Solo y E) Todas

49. La figura muestra un vagn de tren,
dos observadores S y S' , adems 3
situaciones distintas para las
posiciones de un pndulo segn sea el
movimiento del vagn.







. Para el observador S' , la situacin
(1) corresponde a un movimiento
del vagn hacia la derecha con
velocidad constante.
. Para el observador S, la situacin
(2) corresponde al caso cuando el
vagn tiene un MRU.
. Para el observador S, las
situaciones 1 y 3 corresponden al
caso cuando el vagn tiene un
MRUV.
A) FFF B) FVF C) FVV
D) VFV E) VVF

50. Con referencia a las leyes de Kepler
indique cules son las proposiciones
verdaderas (V) o falsas (F).
. En la expresin
2 3
T KR el valor
de K depende de las masas de los
cuerpos que interactan.
. Si el radio medio de la rbita de
Venus alrededor del Sol es
V T
R 0,73R entonces el periodo
del planeta Venus es
V T
T 0, 3T
S
S'
(1)
(2)
(3)
37
CICLO PRLUNIVLRSI1ARIO ADMISIN 2007-I SLMINARIO N 02

CLPRL-UNI ISICA 11
. Kepler demostr que la rbita de
los planetas alrededor del Sol son
elpticas.
A) FFF B) FVF C) FVV
D) FFV E) VFF

51. Seale la verdad (V) o falsedad (F) de
las siguientes proposiciones
. Si dos satlites A y B en rbita
circular a sobre la superficie
de Marte tienen masas

A B
m 3/ 2 m , entonces el periodo
de A es igual al periodo de B.
. Si el radio medio de la rbita de
Venus alrededor del Sol es
V T T
R 0,73R , R radio de la rbita
terrestre, entonces el periodo del
planeta Venus es aproximadamente
227 das terrestres.
. El grfico muestra la trayectoria de
un planeta en rbita alrededor del
Sol. Las rapideces del planeta en A
y B son iguales.






A) VVV B) VVF C) VFF
D) FFF E) FVF

52. En la figura se muestra la rbita
elptica de un planeta que gira
alrededor del Sol, con un periodo igual
a . Si el planeta demora meses
en ir del punto A al perihelio y 2 aos
seis meses en ir del punto B al afelio,
qu fraccin del rea de la elipse es
el rea de la parte sombreada?






A)
1
2
B)
1
3
C)
1
4

D)
1
5
E)
1



53. Si para los planetas orbitando
alrededor del Sol se tiene que T es el
periodo, v

la velocidad de traslacin y
R la distancia al Sol, cules de las
siguientes grficas son correctas?
. .








.



A) y B) y C) y
D) Todas E) Ninguna

54. En un punto sobre su rbita, distante
9
2 10 m L del Sol, el planeta Casiopea
tiene una rapidez angular de
7
2 10 rad/s

L . Cul ser su rapidez


angular (en rad/s) a una distancia de
8
2 10 m L ?
A)

10

B)

2 10

L
C)

3 10

L D)

4 10

L
E)

,3 10

L

55. Dos satlites A y B mantienen
movimientos circulares alrededor de
un mismo planeta, la relacin de radios
de rbita es
A
B
r 4
r 1
, adems el periodo
de B es 27 das. Determine el tiempo
(en horas) en que el satlite A barre
los 2/3 del rea total de su rbita.

B A
B A
v
2

1/R
T
v
3

R
v
2

CICLO PRLUNIVLRSI1ARIO ADMISIN 2007-I SLMINARIO N 02

CLPRL-UNI ISICA 12
A) 88 B) 94 C) 144
D) 182 E) 19

5. Dos satlites A y B giran en forma
circular alrededor de la Tierra. El
satlite A da la vuelta a la Tierra en
y el otro lo realiza en 5 das.
Halle el radio de la rbita del satlite A,
si el radio de la rbita del satlite B es
2R, donde R es el radio de la Tierra.
A) R B) 2R C) 3R
D) 5R E) 7R

57. La figura muestra la trayectoria elptica
de un cometa que se desplaza en
torno al Sol. Si el rea barrida por el
radio vector del cometa entre C y D es
seis veces el rea barrida por el radio
vector entre los puntos A y B, adems
el tiempo transcurrido entre A y B es 3
meses, halle el periodo (en meses) del
cometa.





A) 38 B) 40 C) 42
D) 45 E) 54

58. Suponiendo que el radio de un planeta
sea del radio de la Tierra,
adems suponiendo que la densidad
de la Tierra fuera el doble que la
densidad del planeta. Determine la
aceleracin de la gravedad (en m/s
2
)
en la superficie del planeta

2
T
g 10 m/s . Considere densidad
del planeta: masa del planeta/volumen.
A) 0,5 B) 0,83 C) 1,2
D) 2,32 E) 2,8

59. En la figura se muestra un sistema de
3 masas ubicadas en los vrtices de
un tringulo rectngulo y otra en el
punto medio de la hipotenusa.
Determine la fuerza resultante que
acta sobre la masa ubicada en el
punto medio de la hipotenusa, en
trminos de

2 2
L G m / 2a .











A) L i j

B) L i j


C) L i j

D) L i j


E) 2L i j



0. Halle el valor de la densidad de la
Tierra (en 10
3
kg/m
3
), considerndola
uniforme, adems tome
2
g 9,8 m/s y

T
R ,38 10 m L .
A) 5,1 B) 5,3 C) 5,5
D) 5,7 E) 5,9

1. Considerando la distancia promedio de
la Tierra a la Luna de 385 000 km y la
relacin de sus masas de 81 a 1, a
qu distancia (en km) de la Tierra se
encuentra un satlite ubicado en la
lnea que une de los centros de la
Tierra y la Luna, sabiendo que es
atrado igualmente por ambos.
A) 185 000 B) 220 000 C) 295 000
D) 34 500 E) 32 000

2. Determine a qu altura (en km) sobre
la superficie de un supuesto planeta
cuyo radio es de 2 700 km debe
elevarse un cuerpo para que su peso
disminuya en 19%.
A) 200 B) 300 C) 513
D) 540 E) 1 500

C
A
D B
y(m)
M
m
M
m
45
a 2
x(m)
CICLO PRLUNIVLRSI1ARIO ADMISIN 2007-I SLMINARIO N 02

CLPRL-UNI ISICA 13
3. Una estrella enana tiene radio

E T
R 3 / 2 R (
T
R , radio de la Tierra)
y masa cuya densidad es
E T
100 8 8 ,
donde
T
8 es densidad de la masa de
la Tierra. Halle la relacin
E T
g g de
las aceleraciones de gravedad en las
superficies de la estrella y la Tierra.
A) 120 B) 150 C) 180
D) 200 E) 210

4. Calcule la fuerza (en kN) con que el
planeta Marte atraera una masa de
en la superficie de este planeta.
Suponga que la masa es 1,1 veces la
masa de la Tierra y que su radio es
M T
R 0,5R .
2
g 10 m/s
A) 3,3 B) 4,4 C) 5,5
D) , E) 7,7

5. Si en el sistema mostrado el botecito
se mueve con rapidez constante de
debido a la masa de 20 kg,
determine el trabajo (en kJ) efectuado
por la fuerza de resistencia del agua
sobre el bote durante los 10 primeros
segundos.

A) 4
B) 2
C) 2
D) 4
E) 8

. En la figura se muestra la grfica de la
fuerza versus la posicin. Determine el
trabajo desarrollado (en J) por "F
desde x 0 hasta x 8m .

A) 48,8
B) 38,8
C) 28,8
D) 58,8
E) 17,8

7. Se estira lentamente un resorte hasta
deformarlo 0,1 m realizando un trabajo
de . Determine la magnitud de la
fuerza (en N) necesaria para deformar
el resorte 0,2 m.
A) 100 B) 200 C) 300
D) 400 E) 500

8. La fuerza requerida para estirar un
resorte vara de cero a 80 N cuando se
le extiende moviendo un extremo
12 cm desde su posicin no
deformada (a) Encuentre la constante
del resorte (en N/m) y (b) determine el
trabajo realizado (en J) en extender el
resorte.
A) 221,0 ; 1,2 B) 259,0 ; 1,8
C) 335,0 ; 2,0 D) 41, ; 3,0
E) 7 ; 4,8

9. La velocidad de una bala a la salida
del can de un fusil de 80 cm de
largo, es de 1000 m/s. Si no se
considera el rozamiento y el can se
mantiene horizontal, cul es la fuerza
media (en kN) ejercida por los gases
de la explosin de la plvora durante el
disparo? (masa de la bala = 10 g)
A) 5,50 B) ,25 C) 7,22
D) 8,49 E) 9,81

70. La esferita se suelta en la posicin
mostrada, cul es la mxima altura
(en m) que alcanza respecto al punto
(P) no considere friccin y R = 2,7 m?













20 kg
R
(O)
liso
(P)
F(N)
3
0
x(m)
20
7
8
0
F
CICLO PRLUNIVLRSI1ARIO ADMISIN 2007-I SLMINARIO N 02

CLPRL-UNI ISICA 14
A) 3,2 B) 4,2 C) 4,8
D) 5,0 E) 5,4

71. Pedrito

m se eleva a s
mismo, a velocidad constante, jalando
de una cuerda atada a su cintura y que
pasa por una polea ideal. Usando el
teorema del trabajo y la energa, halle
el trabajo (en kJ) que debe realizar
Pedrito para elevarse 10 m.

A) 0
B) 2
C) 4
D)
E) 8






72. Una bala con una masa de y
una velocidad de 00 m/s penetra un
rbol hasta una distancia de 4,0 cm.
Encuentre la fuerza de friccin
promedio (en kN) que detiene la bala;
suponiendo que la fuerza de friccin es
constante, determine cunto tiempo
(en ms) transcurre entre el momento
que la bola entra en el rbol y el
momento en que se detiene.
A) 11200 ; 10 B) 15000 ; 20
C) 18888 ; 0,2 D) 22,500 ; 0,13
E) 32427 ; 0,08

73. ndique verdadero (V) o falso (F) a las
siguientes proposiciones:
. La condicin para que se conserve
la energa mecnica es que no
existan fuerzas no conservativas.
. Si el efectuado por una fuerza
para mover un cuerpo desde una
posicin inicial a una final es
independiente de la trayectoria,
entonces la fuerza es conservativa.
. Las fuerzas no conservativas son
disipativas.
A) VVV B) VFV C) FVV
D) FVF E) FFF

74. Con respecto a la energa potencial
gravitatoria y la energa potencial
elstica seale verdadero (V) o falso
(F) para las siguientes afirmaciones:
. La energa potencial gravitatoria
aumenta siempre que se hace
trabajo en contra del peso.
. La energa potencial elstica
aumenta siempre que el resorte se
acerca a su posicin de equilibrio.
. La suma de la energa potencial
elstica y gravitatoria siempre se
mantiene constante.
A) VVV B) VFV C) VVF
D) VFF E) FFF

75. Sobre un resorte en posicin vertical
inicialmente no deformado, se coloca
un bloque de masa y se le
suelta. La constante elstica del
resorte es k 100 N/m . Calcule en
metros, la mxima deformacin del
resorte.
2
g 9,8 m/s .







A) 0,198 B) 0,224 C) 0,301
D) 0,392 E) 0,48

7. Un resorte de longitud original,
0
20m y constante
N
K 500
m
,
est unido a un aro que se desliza
sobre una varilla como se muestra en
la figura. Calcule el trabajo realizado
(en J) por la fuerza elstica, cuando el
k
CICLO PRLUNIVLRSI1ARIO ADMISIN 2007-I SLMINARIO N 02

CLPRL-UNI ISICA 15
aro va desde A hasta B. Despreciar el
rozamiento.










A) 141,1 B) 182,5 C) 23,3
D) 282,2 E) 357,5

77. Un bloque de masa m 2kg est
comprimiendo el resorte de constante
k una longitud de . Cuando el
bloque se suelta, desliza sobre la
superficie horizontal lisa y efecta un
movimiento parablico, llegando al
piso con rapidez v m/s . Halle la
constante del resorte
3
en 10 N/m .








A) 40 B) 0 C) 80
D) 100 E) 120

78. El auto de la figura se desliza 35 m
mnimo antes de detenerse cuando
aplica los frenos. Qu distancia en m
se deslizar si su velocidad fuera el
doble?.





A) 140 B) 150 C) 10
D) 170 E) 180
79. La barra homognea de 2 kg desliza
sobre una superficie horizontal lisa,
pero, a partir del punto (A), se vuelve
rugosa. La barra ingresa 1 m de su
longitud antes de detenerse sobre sta
ltima superficie. Determine "v (en
m/s).
2
g 10 m/s .





A) 1 B) 2 C) 2 2
D) 3 E) 4

80. Se suelta un bloquecito de masa
100 g, en el punto A de una cavidad
esfrica rugosa de radio 1 m. El
bloquecito partiendo del reposo se
desliza por la superficie y sube hasta
(y no ms arriba de) el punto B.
Cunto vale el trabajo, en J, realizado
por la fuerza de friccin en el recorrido
de A a B?
2
g 9,8 m/s .








A) 0,49 B) 0,98 C) 4,9
D) 9,8 E) 10,8

81. En la figura, la esferita se suelta desde
la posicin mostrada. Halle H (en m)
para que, luego de recorrer parte del
rizo impacta en el punto (P).
Considere R






53
37
B
A
y
24 cm
k
1 m
B
7
v


v
(A)
0, 3
1,5 m
B
A
R/2
R
R
CICLO PRLUNIVLRSI1ARIO ADMISIN 2007-I SLMINARIO N 02

CLPRL-UNI ISICA 16













A) 4,0 B) 5,5 C) ,4
D) 7,0 E) 8,0

82. Determine la velocidad mnima (en
m/s) que debe tener la partcula de
masa m en la posicin P para llegar
hasta el punto ms alto "Q
considerando que no existe
rozamiento y R 5 m .










A) 3 2 B) 5 2 C) 10
D) 10 2 E) 20 2

83. Una cuerda se desliza hacia la
izquierda del alambre sin friccin.
Qu rapidez debe tener en el punto D
para que se detenga en el punto B?








A)
hg
3
B) 3hg C)
2
hg
3

D)
3
hg
2
E)
4
hg
3


84. La figura muestra un carril de
que se puede deslizar sin friccin
sobre la varilla horizontal. Si el carril se
une a un resorte de constante
k 4 N/m y longitud natural 5 m, halle
la mxima rapidez (en m/s) que
adquiere si parte del reposo.









A) 2 B) 4 C)
D) 8 E) 10

85. Con un bloque de 1 kg se comprime
un resorte de k 400 N/m y se
deforma 0,2 m como se muestra en la
figura. Al liberarlo recorre por la pista
ABC de
c
0,5 3 ; L ,25 m y
53 7 . Si en C se detiene, halle la
distancia entre B y C (en m).

2
g 10 m/s








A) 2,25
B) 3,45
C) 5,50
D) 7,85
E) 8,85
R
liso
(P)
H
Q
R
P
m
h
A

2h
3
h
h
2

h
3

B
C

D

24 m
7

m

7
d
C
A
B
L
g


CICLO PRLUNIVLRSI1ARIO ADMISIN 2007-I SLMINARIO N 02

CLPRL-UNI ISICA 17
8. En la figura, la esfera es de de
masa y la cuerda de masa
insignificante y 5 m de longitud se
sueltan desde la posicin mostrada.
Determine la tensin en la cuerda (en
N) en el instante en que la rapidez de
la esfera es la mitad de su rapidez
mxima (g = 10 m/s
2
).





A) 5 B) 10 C) 15
D) 20 E) 25

87. Para poner en funcionamiento una
excavadora se usa un motor que
ejerce una fuerza variable en el
tiempo, segn se ilustra en la grfica.
Asumiendo que la masa del dispositivo
consumidor de potencia es de 324 kg,
determine la potencia (en k)
entregada por el motor durante los
100 s de su funcionamiento.









A) 5 B) 10 C) 15
D) 20 E) 25

88. Un bloque de se desplaza 8 m,
a partir del reposo, debido a la accin
de una fuerza horizontal constante de
200 N, en un piso horizontal rugoso
que produce una fuerza de friccin de
40 N. Calcule la potencia utilizada para
mover al bloque (en k).
A) 1,0 B) 1,13 C) 2,10
D) 2,50 E) 3,50
89. Sobre un bloque de 1,5 kg,
inicialmente movindose con 80i

m/s,
acta una fuerza de 12i 15 j

N
durante 20 s. Si el bloque slo puede
moverse a lo largo del eje x, halle la
potencia entregada al bloque (en k).
A) 0,52 B) 1,92 C) 3,52
D) 4,35 E) 5,20

90. Sobre un bloque de 5 kg, inicialmente
en reposo sobre una superficie
horizontal lisa, se aplica una fuerza
tambin horizontal de 10 N. Halle la
potencia media (en ) desarrollada
por la fuerza durante los 3 primeros
segundos y la potencia instantnea (en
) en t 2s .
A) 30 ; 40 B) 30 ; 30 C) 40 ; 40
D) 35 ; 35 E) 20 ; 30

91. Un nio de de masa se
suspende de la cuerda mostrada en la
figura la cual est unidad al bloque A
de 5 kg; este a su vez unido mediante
otra cuerda al bloque B de 7 kg.
Determine la potencia desarrollada (en
) por el peso del nio hasta llegar a
Tierra.














A) 250 2 B) 500 2
C) 800 D) 400
E) 750 2
F(N)
t(s)
10
500
200
50 100
0
A
B
50 m
g

S-ar putea să vă placă și